Asymptotic of sum $sum_{j=1}^n j^{f(n)}$












4












$begingroup$


What is known about the asymptotic of $sum_{j=1}^n j^{f(n)}$ where the exponent is some function that grows with $n$? For instance, if $f(n) = k$ is constant, then we know it's $frac{1}{k+1}n^{k+1} + O(n^k)$. If $f(n) = n$, it seems that the sum is dominated by the last few terms and behaves like a geometric series with $r=1/e$, so that the sum grows as $n^nfrac{e}{e-1}$ (plus some error term). What happens if e.g. $f(n) = n^alpha$ for $0 < alpha < 1$ or $f(n) = log n$?










share|cite|improve this question











$endgroup$












  • $begingroup$
    do you mean f(n) or f(j) in the exponent?
    $endgroup$
    – Sandeep Silwal
    Dec 4 '18 at 1:47
















4












$begingroup$


What is known about the asymptotic of $sum_{j=1}^n j^{f(n)}$ where the exponent is some function that grows with $n$? For instance, if $f(n) = k$ is constant, then we know it's $frac{1}{k+1}n^{k+1} + O(n^k)$. If $f(n) = n$, it seems that the sum is dominated by the last few terms and behaves like a geometric series with $r=1/e$, so that the sum grows as $n^nfrac{e}{e-1}$ (plus some error term). What happens if e.g. $f(n) = n^alpha$ for $0 < alpha < 1$ or $f(n) = log n$?










share|cite|improve this question











$endgroup$












  • $begingroup$
    do you mean f(n) or f(j) in the exponent?
    $endgroup$
    – Sandeep Silwal
    Dec 4 '18 at 1:47














4












4








4


0



$begingroup$


What is known about the asymptotic of $sum_{j=1}^n j^{f(n)}$ where the exponent is some function that grows with $n$? For instance, if $f(n) = k$ is constant, then we know it's $frac{1}{k+1}n^{k+1} + O(n^k)$. If $f(n) = n$, it seems that the sum is dominated by the last few terms and behaves like a geometric series with $r=1/e$, so that the sum grows as $n^nfrac{e}{e-1}$ (plus some error term). What happens if e.g. $f(n) = n^alpha$ for $0 < alpha < 1$ or $f(n) = log n$?










share|cite|improve this question











$endgroup$




What is known about the asymptotic of $sum_{j=1}^n j^{f(n)}$ where the exponent is some function that grows with $n$? For instance, if $f(n) = k$ is constant, then we know it's $frac{1}{k+1}n^{k+1} + O(n^k)$. If $f(n) = n$, it seems that the sum is dominated by the last few terms and behaves like a geometric series with $r=1/e$, so that the sum grows as $n^nfrac{e}{e-1}$ (plus some error term). What happens if e.g. $f(n) = n^alpha$ for $0 < alpha < 1$ or $f(n) = log n$?







real-analysis limits analysis summation asymptotics






share|cite|improve this question















share|cite|improve this question













share|cite|improve this question




share|cite|improve this question








edited Dec 3 '18 at 23:54







maridia

















asked Dec 3 '18 at 22:37









maridiamaridia

1,065113




1,065113












  • $begingroup$
    do you mean f(n) or f(j) in the exponent?
    $endgroup$
    – Sandeep Silwal
    Dec 4 '18 at 1:47


















  • $begingroup$
    do you mean f(n) or f(j) in the exponent?
    $endgroup$
    – Sandeep Silwal
    Dec 4 '18 at 1:47
















$begingroup$
do you mean f(n) or f(j) in the exponent?
$endgroup$
– Sandeep Silwal
Dec 4 '18 at 1:47




$begingroup$
do you mean f(n) or f(j) in the exponent?
$endgroup$
– Sandeep Silwal
Dec 4 '18 at 1:47










2 Answers
2






active

oldest

votes


















2












$begingroup$

(Note the calculations in this post might be off). This is a perfect example of the Euler-Maclaurin formula which states that:



$$ sum_{j=1}^n f(j) sim int_1^n f(x) dx + frac{f(1)+f(n)}2 + sum_{k=1}^{infty} frac{B_{2k}}{(2k)!}(f^{(2k-1)}(n) - f^{(2k-1)}(1)).$$



For more information, see here. If we use this formula for the function $f(x) = x^{log(n)}$, we get the following bound:
$$ sum_{j=1}^n j^{log(n)} = frac{n^{log(n) + 1}}{log(n) + 1} + frac{n^{log(n)}}2 + O(log(n) n^{log(n)-1}). $$ I think all the other cases can be handled similarly assuming $f$ is a sufficiently nice function.






share|cite|improve this answer











$endgroup$













  • $begingroup$
    hi Sandeep, I think the last term should be $(log n) n^{log (n) - 1}$?
    $endgroup$
    – maridia
    Dec 4 '18 at 4:09












  • $begingroup$
    Thanks, this formula seems to give a non-trivial expansion exactly when $f(n) = o(n)$. When $f(n)$ is on order $n$, I think the geometric series behavior is correct and for superlinear growth, I think the entire sum is dominated by the last term.
    $endgroup$
    – maridia
    Dec 4 '18 at 4:10












  • $begingroup$
    @norfair yea i think you are correct
    $endgroup$
    – Sandeep Silwal
    Dec 4 '18 at 5:29



















1












$begingroup$

(I have now rewritten this 3 times and, though another answer has been given in the meantime, I'll still submit it.)



If you are to use the Euler-Maclaurin formula you need to be careful with the remainder term, ensuring it doesn't blow up.



Let $g(x) = x^{f(n)}$, then we can write



begin{align}
F_f(n)
&equiv sum_{j=1}^n j^{f(n)}
= sum_{j=1}^n g(j)\
&= underbrace{int_0^n g(x);dx}_{(1)} + underbrace{sum_{k=1}^p frac{B_k}{k!}left(g^{(k-1)}(n) - g^{(k-1)}(0)right)}_{(2)} +underbrace{ R_p}_{(3)}
end{align}



(1)
$$int_0^n g(x);dx = int_0^n x^{f(n)};dx = frac{n^{f(n)+1}}{f(n)+1}$$



(2)
We can inductively show that $g^{(k)}(x) = P_k(n)x^{f(n)-k}$ for a sequence $big(P_k(n)big)_k$.
Clearly $g(x)=g^{(0)}(x)=P_0(n)x^{f(n) - 0}$ where $P_0(n)= 1$, and
begin{align}
g^{(k)}(x) &= frac{d}{dx} g^{(k-1)}(x)
= frac{d}{dx} P_{k-1}(n)x^{f(n)-(k-1)}\
&= P_{k-1}(n)big(f(n)-k+1big) x^{f(n)-(k-1)-1}
= P_k(n) x^{f(n)-k}
end{align}

where $P_k(n) equiv P_{k-1}(n)big(f(n)-k+1big)$. We can compute
$$P_k(n) = P_{k-1}(n)big(f(n)-k+1big) = prod_{j=0}^{k-1} big(f(n)-jbig)$$



Hence
begin{align}
sum_{k=1}^p frac{B_k}{k!}left(g^{(k-1)}(n) - g^{(k-1)}(0)right)
&= sum_{k=1}^p frac{B_k}{k!}left(P^{(k-1)}(n)n^{f(n)-k+1} - P^{(k-1)}(n)cdot 0right)\
&= sum_{k=1}^p frac{B_k}{k!}n^{f(n)-k+1} cdotprod_{j=0}^{k-2}big(f(n)-jbig)
end{align}



(3) We have the following bound on the remainder term:
begin{align}
|R_p|
&leq frac{2zeta(p )}{(2pi)^p} int_0^n |g^{(p )}(x)|;dx
= frac{2zeta(p )}{(2pi)^p} int_0^n |P_p(n)x^{f(n)-p}|;dx\
&overset{(*)}{=} frac{2zeta(p )}{(2pi)^p} |P_p(n)| frac{n^{f(n)-p+1}}{f(n)-p+1}
= frac{2zeta(p )}{(2pi)^p} prod_{j=0}^{p-2} big(f(n)-jbig) n^{f(n)-p+1}
end{align}



at $(*)$ we require $f(n)geq p$ for the integral to converge.



In particular, if we only take 2 terms from (2) (which we do to avoid $zeta(1)$), then we bound the



begin{align}
|R_2|
&leq frac{2zeta(2)}{(2pi)^2} prod_{j=0}^{2-2} big(f(n)-jbig) n^{f(n)-2+1}
= frac{2(pi^2/6)}{4pi^2} f(n) n^{f(n)-1}\
%
&= frac{1}{12} f(n) n^{f(n)-1}
end{align}



With $p=2$ we then write
begin{align}
F_f(n)
%
&= frac{n^{f(n)+1}}{f(n)+1} + sum_{k=1}^2 frac{B_k}{k!}n^{f(n)-k+1} cdotprod_{j=0}^{k-2}big(f(n)-jbig) + R_2\
%
&= frac{n^{f(n)+1}}{f(n)+1} + frac{(1 /2)}{1!}n^{f(n)-1+1} + frac{(1 /6)}{2!}n^{f(n)-2+1}f(n) + R_2\
%
&= n^{f(n)}left[frac{n}{f(n)+1} + frac{1}{2} + frac{1}{12}frac{f(n)}{n}right] + R_2\
end{align}



Example 1
For $f(n)=n$:
$$F_f(n)
= n^nleft[frac{n}{n+1} + frac{1}{2} + frac{1}{12}frac{n}{n}right] + R_2
= n^n left[frac{19}{12} - frac{1}{n+1}right] + R_2$$



$$|R_2| leq frac{1}{12} ncdot n^{n-1} = frac{1}{12} n^n$$
so
$$lim_{ntoinfty} frac{F_f(n)}{n^n} in left(tfrac{19}{12}-tfrac{1}{12}, tfrac{19}{12} + tfrac{1}{12}right) = left(tfrac{3}{2}, tfrac{5}{3}right)$$



Indeed, a check:
$$frac{e}{e-1} in (tfrac{3}{2}, tfrac{5}{3})$$



Example 2
For $f(n)=log(n)$:
begin{align}
F_f(n)
%
&= n^{log(n)}left[frac{n}{log(n)+1} + frac{1}{2} + frac{1}{12}frac{log(n)}{n}right] + R_2\
%
&= frac{n^{log(n)+1}}{log(n)}left[frac{log(n)}{log(n)+1} + frac{1}{2}frac{log(n)}{n} + frac{1}{12}left(frac{log(n)}{n}right)^2right] + R_2\
end{align}



begin{align}
|R_2|
&leq frac{1}{12} f(n) n^{f(n)-1}
= frac{1}{12} log(n) n^{log(n)-1}\
%
&= frac{1}{12} frac{n^{log(n)+1}}{log(n)} left(frac{log(n)}{n}right)^2
end{align}



So
$$lim_{ntoinfty} frac{F_f(n)}{frac{n^{log(n)+1}}{log(n)}} = 1$$
because $frac{|R_2|}{frac{n^{log(n)+1}}{log(n)}} = left(frac{log(n)}{n}right)^2 to 0$.



Example 3
$f(n) = n^alpha$ for $alphain(0,1)$:



begin{align}
F_f(n)
%
&=frac{n^{n^alpha+1}}{n^alpha+1} + frac{1}{2}n^{n^alpha-1+1} + frac{1}{12}n^{n^alpha-1}n^alpha + R_2\
%
&=frac{n^{alpha}}{n^alpha+1}n^{n^alpha+(1-alpha)} + frac{1}{2}n^{n^alpha} + frac{1}{12}n^{n^alpha-(1-alpha)} + R_2\
end{align}



$$|R_2|
%
leq frac{1}{12} f(n) n^{f(n)-1}
= frac{1}{12} n^{n^alpha-(1-alpha)}$$



$$lim_{ntoinfty} frac{F_f(n)}{n^{n^alpha+(1-alpha)}} = 1$$
as $$frac{|R_2|}{n^{n^alpha+(1-alpha)}} = frac{1}{12 n^{2(1-alpha)}} to 0$$



Example 4
For $f(n)=n^beta$, $beta > 1$:



begin{align}
F_f(n)
&=frac{n^{f(n)+1}}{f(n)}frac{f(n)}{f(n)+1} + frac{1}{2}n^{f(n)} + frac{1}{12}n^{f(n)-1}f(n) + R_2\
&= n^{n^beta - (beta - 1)} frac{n^beta}{n^beta+1} + frac{1}{2}n^{n^beta} + frac{1}{12}n^{n^beta+(beta-1)} + R_2
end{align}



$$|R_2|
leq frac{1}{12} n^beta n^{n^beta-1}
= frac{1}{12} n^{n^beta+(beta-1)}
$$



Hence
$$lim_{ntoinfty} frac{F_f(n)}{n^{n^beta+(beta-1)}}
in left(tfrac{1}{12} - tfrac{1}{12}, tfrac{1}{12} + tfrac{1}{12}right) = left(0, tfrac{1}{6}right)$$






share|cite|improve this answer











$endgroup$













  • $begingroup$
    Hi adfriedman, thank you for the detailed answer. Regarding your Example 4, I'm not sure you can conclude that the limit is in the open interval $(0, 1/6)$. In fact, I believe the limit will be 0 since heuristically it seems that $lim_{n to infty} F_f(n)/n^{n^{beta}} = 1$. In other words, the sum is dominated by just the last term.
    $endgroup$
    – maridia
    Dec 4 '18 at 15:59













Your Answer





StackExchange.ifUsing("editor", function () {
return StackExchange.using("mathjaxEditing", function () {
StackExchange.MarkdownEditor.creationCallbacks.add(function (editor, postfix) {
StackExchange.mathjaxEditing.prepareWmdForMathJax(editor, postfix, [["$", "$"], ["\\(","\\)"]]);
});
});
}, "mathjax-editing");

StackExchange.ready(function() {
var channelOptions = {
tags: "".split(" "),
id: "69"
};
initTagRenderer("".split(" "), "".split(" "), channelOptions);

StackExchange.using("externalEditor", function() {
// Have to fire editor after snippets, if snippets enabled
if (StackExchange.settings.snippets.snippetsEnabled) {
StackExchange.using("snippets", function() {
createEditor();
});
}
else {
createEditor();
}
});

function createEditor() {
StackExchange.prepareEditor({
heartbeatType: 'answer',
autoActivateHeartbeat: false,
convertImagesToLinks: true,
noModals: true,
showLowRepImageUploadWarning: true,
reputationToPostImages: 10,
bindNavPrevention: true,
postfix: "",
imageUploader: {
brandingHtml: "Powered by u003ca class="icon-imgur-white" href="https://imgur.com/"u003eu003c/au003e",
contentPolicyHtml: "User contributions licensed under u003ca href="https://creativecommons.org/licenses/by-sa/3.0/"u003ecc by-sa 3.0 with attribution requiredu003c/au003e u003ca href="https://stackoverflow.com/legal/content-policy"u003e(content policy)u003c/au003e",
allowUrls: true
},
noCode: true, onDemand: true,
discardSelector: ".discard-answer"
,immediatelyShowMarkdownHelp:true
});


}
});














draft saved

draft discarded


















StackExchange.ready(
function () {
StackExchange.openid.initPostLogin('.new-post-login', 'https%3a%2f%2fmath.stackexchange.com%2fquestions%2f3024800%2fasymptotic-of-sum-sum-j-1n-jfn%23new-answer', 'question_page');
}
);

Post as a guest















Required, but never shown

























2 Answers
2






active

oldest

votes








2 Answers
2






active

oldest

votes









active

oldest

votes






active

oldest

votes









2












$begingroup$

(Note the calculations in this post might be off). This is a perfect example of the Euler-Maclaurin formula which states that:



$$ sum_{j=1}^n f(j) sim int_1^n f(x) dx + frac{f(1)+f(n)}2 + sum_{k=1}^{infty} frac{B_{2k}}{(2k)!}(f^{(2k-1)}(n) - f^{(2k-1)}(1)).$$



For more information, see here. If we use this formula for the function $f(x) = x^{log(n)}$, we get the following bound:
$$ sum_{j=1}^n j^{log(n)} = frac{n^{log(n) + 1}}{log(n) + 1} + frac{n^{log(n)}}2 + O(log(n) n^{log(n)-1}). $$ I think all the other cases can be handled similarly assuming $f$ is a sufficiently nice function.






share|cite|improve this answer











$endgroup$













  • $begingroup$
    hi Sandeep, I think the last term should be $(log n) n^{log (n) - 1}$?
    $endgroup$
    – maridia
    Dec 4 '18 at 4:09












  • $begingroup$
    Thanks, this formula seems to give a non-trivial expansion exactly when $f(n) = o(n)$. When $f(n)$ is on order $n$, I think the geometric series behavior is correct and for superlinear growth, I think the entire sum is dominated by the last term.
    $endgroup$
    – maridia
    Dec 4 '18 at 4:10












  • $begingroup$
    @norfair yea i think you are correct
    $endgroup$
    – Sandeep Silwal
    Dec 4 '18 at 5:29
















2












$begingroup$

(Note the calculations in this post might be off). This is a perfect example of the Euler-Maclaurin formula which states that:



$$ sum_{j=1}^n f(j) sim int_1^n f(x) dx + frac{f(1)+f(n)}2 + sum_{k=1}^{infty} frac{B_{2k}}{(2k)!}(f^{(2k-1)}(n) - f^{(2k-1)}(1)).$$



For more information, see here. If we use this formula for the function $f(x) = x^{log(n)}$, we get the following bound:
$$ sum_{j=1}^n j^{log(n)} = frac{n^{log(n) + 1}}{log(n) + 1} + frac{n^{log(n)}}2 + O(log(n) n^{log(n)-1}). $$ I think all the other cases can be handled similarly assuming $f$ is a sufficiently nice function.






share|cite|improve this answer











$endgroup$













  • $begingroup$
    hi Sandeep, I think the last term should be $(log n) n^{log (n) - 1}$?
    $endgroup$
    – maridia
    Dec 4 '18 at 4:09












  • $begingroup$
    Thanks, this formula seems to give a non-trivial expansion exactly when $f(n) = o(n)$. When $f(n)$ is on order $n$, I think the geometric series behavior is correct and for superlinear growth, I think the entire sum is dominated by the last term.
    $endgroup$
    – maridia
    Dec 4 '18 at 4:10












  • $begingroup$
    @norfair yea i think you are correct
    $endgroup$
    – Sandeep Silwal
    Dec 4 '18 at 5:29














2












2








2





$begingroup$

(Note the calculations in this post might be off). This is a perfect example of the Euler-Maclaurin formula which states that:



$$ sum_{j=1}^n f(j) sim int_1^n f(x) dx + frac{f(1)+f(n)}2 + sum_{k=1}^{infty} frac{B_{2k}}{(2k)!}(f^{(2k-1)}(n) - f^{(2k-1)}(1)).$$



For more information, see here. If we use this formula for the function $f(x) = x^{log(n)}$, we get the following bound:
$$ sum_{j=1}^n j^{log(n)} = frac{n^{log(n) + 1}}{log(n) + 1} + frac{n^{log(n)}}2 + O(log(n) n^{log(n)-1}). $$ I think all the other cases can be handled similarly assuming $f$ is a sufficiently nice function.






share|cite|improve this answer











$endgroup$



(Note the calculations in this post might be off). This is a perfect example of the Euler-Maclaurin formula which states that:



$$ sum_{j=1}^n f(j) sim int_1^n f(x) dx + frac{f(1)+f(n)}2 + sum_{k=1}^{infty} frac{B_{2k}}{(2k)!}(f^{(2k-1)}(n) - f^{(2k-1)}(1)).$$



For more information, see here. If we use this formula for the function $f(x) = x^{log(n)}$, we get the following bound:
$$ sum_{j=1}^n j^{log(n)} = frac{n^{log(n) + 1}}{log(n) + 1} + frac{n^{log(n)}}2 + O(log(n) n^{log(n)-1}). $$ I think all the other cases can be handled similarly assuming $f$ is a sufficiently nice function.







share|cite|improve this answer














share|cite|improve this answer



share|cite|improve this answer








edited Dec 4 '18 at 5:29

























answered Dec 4 '18 at 2:22









Sandeep SilwalSandeep Silwal

5,84311236




5,84311236












  • $begingroup$
    hi Sandeep, I think the last term should be $(log n) n^{log (n) - 1}$?
    $endgroup$
    – maridia
    Dec 4 '18 at 4:09












  • $begingroup$
    Thanks, this formula seems to give a non-trivial expansion exactly when $f(n) = o(n)$. When $f(n)$ is on order $n$, I think the geometric series behavior is correct and for superlinear growth, I think the entire sum is dominated by the last term.
    $endgroup$
    – maridia
    Dec 4 '18 at 4:10












  • $begingroup$
    @norfair yea i think you are correct
    $endgroup$
    – Sandeep Silwal
    Dec 4 '18 at 5:29


















  • $begingroup$
    hi Sandeep, I think the last term should be $(log n) n^{log (n) - 1}$?
    $endgroup$
    – maridia
    Dec 4 '18 at 4:09












  • $begingroup$
    Thanks, this formula seems to give a non-trivial expansion exactly when $f(n) = o(n)$. When $f(n)$ is on order $n$, I think the geometric series behavior is correct and for superlinear growth, I think the entire sum is dominated by the last term.
    $endgroup$
    – maridia
    Dec 4 '18 at 4:10












  • $begingroup$
    @norfair yea i think you are correct
    $endgroup$
    – Sandeep Silwal
    Dec 4 '18 at 5:29
















$begingroup$
hi Sandeep, I think the last term should be $(log n) n^{log (n) - 1}$?
$endgroup$
– maridia
Dec 4 '18 at 4:09






$begingroup$
hi Sandeep, I think the last term should be $(log n) n^{log (n) - 1}$?
$endgroup$
– maridia
Dec 4 '18 at 4:09














$begingroup$
Thanks, this formula seems to give a non-trivial expansion exactly when $f(n) = o(n)$. When $f(n)$ is on order $n$, I think the geometric series behavior is correct and for superlinear growth, I think the entire sum is dominated by the last term.
$endgroup$
– maridia
Dec 4 '18 at 4:10






$begingroup$
Thanks, this formula seems to give a non-trivial expansion exactly when $f(n) = o(n)$. When $f(n)$ is on order $n$, I think the geometric series behavior is correct and for superlinear growth, I think the entire sum is dominated by the last term.
$endgroup$
– maridia
Dec 4 '18 at 4:10














$begingroup$
@norfair yea i think you are correct
$endgroup$
– Sandeep Silwal
Dec 4 '18 at 5:29




$begingroup$
@norfair yea i think you are correct
$endgroup$
– Sandeep Silwal
Dec 4 '18 at 5:29











1












$begingroup$

(I have now rewritten this 3 times and, though another answer has been given in the meantime, I'll still submit it.)



If you are to use the Euler-Maclaurin formula you need to be careful with the remainder term, ensuring it doesn't blow up.



Let $g(x) = x^{f(n)}$, then we can write



begin{align}
F_f(n)
&equiv sum_{j=1}^n j^{f(n)}
= sum_{j=1}^n g(j)\
&= underbrace{int_0^n g(x);dx}_{(1)} + underbrace{sum_{k=1}^p frac{B_k}{k!}left(g^{(k-1)}(n) - g^{(k-1)}(0)right)}_{(2)} +underbrace{ R_p}_{(3)}
end{align}



(1)
$$int_0^n g(x);dx = int_0^n x^{f(n)};dx = frac{n^{f(n)+1}}{f(n)+1}$$



(2)
We can inductively show that $g^{(k)}(x) = P_k(n)x^{f(n)-k}$ for a sequence $big(P_k(n)big)_k$.
Clearly $g(x)=g^{(0)}(x)=P_0(n)x^{f(n) - 0}$ where $P_0(n)= 1$, and
begin{align}
g^{(k)}(x) &= frac{d}{dx} g^{(k-1)}(x)
= frac{d}{dx} P_{k-1}(n)x^{f(n)-(k-1)}\
&= P_{k-1}(n)big(f(n)-k+1big) x^{f(n)-(k-1)-1}
= P_k(n) x^{f(n)-k}
end{align}

where $P_k(n) equiv P_{k-1}(n)big(f(n)-k+1big)$. We can compute
$$P_k(n) = P_{k-1}(n)big(f(n)-k+1big) = prod_{j=0}^{k-1} big(f(n)-jbig)$$



Hence
begin{align}
sum_{k=1}^p frac{B_k}{k!}left(g^{(k-1)}(n) - g^{(k-1)}(0)right)
&= sum_{k=1}^p frac{B_k}{k!}left(P^{(k-1)}(n)n^{f(n)-k+1} - P^{(k-1)}(n)cdot 0right)\
&= sum_{k=1}^p frac{B_k}{k!}n^{f(n)-k+1} cdotprod_{j=0}^{k-2}big(f(n)-jbig)
end{align}



(3) We have the following bound on the remainder term:
begin{align}
|R_p|
&leq frac{2zeta(p )}{(2pi)^p} int_0^n |g^{(p )}(x)|;dx
= frac{2zeta(p )}{(2pi)^p} int_0^n |P_p(n)x^{f(n)-p}|;dx\
&overset{(*)}{=} frac{2zeta(p )}{(2pi)^p} |P_p(n)| frac{n^{f(n)-p+1}}{f(n)-p+1}
= frac{2zeta(p )}{(2pi)^p} prod_{j=0}^{p-2} big(f(n)-jbig) n^{f(n)-p+1}
end{align}



at $(*)$ we require $f(n)geq p$ for the integral to converge.



In particular, if we only take 2 terms from (2) (which we do to avoid $zeta(1)$), then we bound the



begin{align}
|R_2|
&leq frac{2zeta(2)}{(2pi)^2} prod_{j=0}^{2-2} big(f(n)-jbig) n^{f(n)-2+1}
= frac{2(pi^2/6)}{4pi^2} f(n) n^{f(n)-1}\
%
&= frac{1}{12} f(n) n^{f(n)-1}
end{align}



With $p=2$ we then write
begin{align}
F_f(n)
%
&= frac{n^{f(n)+1}}{f(n)+1} + sum_{k=1}^2 frac{B_k}{k!}n^{f(n)-k+1} cdotprod_{j=0}^{k-2}big(f(n)-jbig) + R_2\
%
&= frac{n^{f(n)+1}}{f(n)+1} + frac{(1 /2)}{1!}n^{f(n)-1+1} + frac{(1 /6)}{2!}n^{f(n)-2+1}f(n) + R_2\
%
&= n^{f(n)}left[frac{n}{f(n)+1} + frac{1}{2} + frac{1}{12}frac{f(n)}{n}right] + R_2\
end{align}



Example 1
For $f(n)=n$:
$$F_f(n)
= n^nleft[frac{n}{n+1} + frac{1}{2} + frac{1}{12}frac{n}{n}right] + R_2
= n^n left[frac{19}{12} - frac{1}{n+1}right] + R_2$$



$$|R_2| leq frac{1}{12} ncdot n^{n-1} = frac{1}{12} n^n$$
so
$$lim_{ntoinfty} frac{F_f(n)}{n^n} in left(tfrac{19}{12}-tfrac{1}{12}, tfrac{19}{12} + tfrac{1}{12}right) = left(tfrac{3}{2}, tfrac{5}{3}right)$$



Indeed, a check:
$$frac{e}{e-1} in (tfrac{3}{2}, tfrac{5}{3})$$



Example 2
For $f(n)=log(n)$:
begin{align}
F_f(n)
%
&= n^{log(n)}left[frac{n}{log(n)+1} + frac{1}{2} + frac{1}{12}frac{log(n)}{n}right] + R_2\
%
&= frac{n^{log(n)+1}}{log(n)}left[frac{log(n)}{log(n)+1} + frac{1}{2}frac{log(n)}{n} + frac{1}{12}left(frac{log(n)}{n}right)^2right] + R_2\
end{align}



begin{align}
|R_2|
&leq frac{1}{12} f(n) n^{f(n)-1}
= frac{1}{12} log(n) n^{log(n)-1}\
%
&= frac{1}{12} frac{n^{log(n)+1}}{log(n)} left(frac{log(n)}{n}right)^2
end{align}



So
$$lim_{ntoinfty} frac{F_f(n)}{frac{n^{log(n)+1}}{log(n)}} = 1$$
because $frac{|R_2|}{frac{n^{log(n)+1}}{log(n)}} = left(frac{log(n)}{n}right)^2 to 0$.



Example 3
$f(n) = n^alpha$ for $alphain(0,1)$:



begin{align}
F_f(n)
%
&=frac{n^{n^alpha+1}}{n^alpha+1} + frac{1}{2}n^{n^alpha-1+1} + frac{1}{12}n^{n^alpha-1}n^alpha + R_2\
%
&=frac{n^{alpha}}{n^alpha+1}n^{n^alpha+(1-alpha)} + frac{1}{2}n^{n^alpha} + frac{1}{12}n^{n^alpha-(1-alpha)} + R_2\
end{align}



$$|R_2|
%
leq frac{1}{12} f(n) n^{f(n)-1}
= frac{1}{12} n^{n^alpha-(1-alpha)}$$



$$lim_{ntoinfty} frac{F_f(n)}{n^{n^alpha+(1-alpha)}} = 1$$
as $$frac{|R_2|}{n^{n^alpha+(1-alpha)}} = frac{1}{12 n^{2(1-alpha)}} to 0$$



Example 4
For $f(n)=n^beta$, $beta > 1$:



begin{align}
F_f(n)
&=frac{n^{f(n)+1}}{f(n)}frac{f(n)}{f(n)+1} + frac{1}{2}n^{f(n)} + frac{1}{12}n^{f(n)-1}f(n) + R_2\
&= n^{n^beta - (beta - 1)} frac{n^beta}{n^beta+1} + frac{1}{2}n^{n^beta} + frac{1}{12}n^{n^beta+(beta-1)} + R_2
end{align}



$$|R_2|
leq frac{1}{12} n^beta n^{n^beta-1}
= frac{1}{12} n^{n^beta+(beta-1)}
$$



Hence
$$lim_{ntoinfty} frac{F_f(n)}{n^{n^beta+(beta-1)}}
in left(tfrac{1}{12} - tfrac{1}{12}, tfrac{1}{12} + tfrac{1}{12}right) = left(0, tfrac{1}{6}right)$$






share|cite|improve this answer











$endgroup$













  • $begingroup$
    Hi adfriedman, thank you for the detailed answer. Regarding your Example 4, I'm not sure you can conclude that the limit is in the open interval $(0, 1/6)$. In fact, I believe the limit will be 0 since heuristically it seems that $lim_{n to infty} F_f(n)/n^{n^{beta}} = 1$. In other words, the sum is dominated by just the last term.
    $endgroup$
    – maridia
    Dec 4 '18 at 15:59


















1












$begingroup$

(I have now rewritten this 3 times and, though another answer has been given in the meantime, I'll still submit it.)



If you are to use the Euler-Maclaurin formula you need to be careful with the remainder term, ensuring it doesn't blow up.



Let $g(x) = x^{f(n)}$, then we can write



begin{align}
F_f(n)
&equiv sum_{j=1}^n j^{f(n)}
= sum_{j=1}^n g(j)\
&= underbrace{int_0^n g(x);dx}_{(1)} + underbrace{sum_{k=1}^p frac{B_k}{k!}left(g^{(k-1)}(n) - g^{(k-1)}(0)right)}_{(2)} +underbrace{ R_p}_{(3)}
end{align}



(1)
$$int_0^n g(x);dx = int_0^n x^{f(n)};dx = frac{n^{f(n)+1}}{f(n)+1}$$



(2)
We can inductively show that $g^{(k)}(x) = P_k(n)x^{f(n)-k}$ for a sequence $big(P_k(n)big)_k$.
Clearly $g(x)=g^{(0)}(x)=P_0(n)x^{f(n) - 0}$ where $P_0(n)= 1$, and
begin{align}
g^{(k)}(x) &= frac{d}{dx} g^{(k-1)}(x)
= frac{d}{dx} P_{k-1}(n)x^{f(n)-(k-1)}\
&= P_{k-1}(n)big(f(n)-k+1big) x^{f(n)-(k-1)-1}
= P_k(n) x^{f(n)-k}
end{align}

where $P_k(n) equiv P_{k-1}(n)big(f(n)-k+1big)$. We can compute
$$P_k(n) = P_{k-1}(n)big(f(n)-k+1big) = prod_{j=0}^{k-1} big(f(n)-jbig)$$



Hence
begin{align}
sum_{k=1}^p frac{B_k}{k!}left(g^{(k-1)}(n) - g^{(k-1)}(0)right)
&= sum_{k=1}^p frac{B_k}{k!}left(P^{(k-1)}(n)n^{f(n)-k+1} - P^{(k-1)}(n)cdot 0right)\
&= sum_{k=1}^p frac{B_k}{k!}n^{f(n)-k+1} cdotprod_{j=0}^{k-2}big(f(n)-jbig)
end{align}



(3) We have the following bound on the remainder term:
begin{align}
|R_p|
&leq frac{2zeta(p )}{(2pi)^p} int_0^n |g^{(p )}(x)|;dx
= frac{2zeta(p )}{(2pi)^p} int_0^n |P_p(n)x^{f(n)-p}|;dx\
&overset{(*)}{=} frac{2zeta(p )}{(2pi)^p} |P_p(n)| frac{n^{f(n)-p+1}}{f(n)-p+1}
= frac{2zeta(p )}{(2pi)^p} prod_{j=0}^{p-2} big(f(n)-jbig) n^{f(n)-p+1}
end{align}



at $(*)$ we require $f(n)geq p$ for the integral to converge.



In particular, if we only take 2 terms from (2) (which we do to avoid $zeta(1)$), then we bound the



begin{align}
|R_2|
&leq frac{2zeta(2)}{(2pi)^2} prod_{j=0}^{2-2} big(f(n)-jbig) n^{f(n)-2+1}
= frac{2(pi^2/6)}{4pi^2} f(n) n^{f(n)-1}\
%
&= frac{1}{12} f(n) n^{f(n)-1}
end{align}



With $p=2$ we then write
begin{align}
F_f(n)
%
&= frac{n^{f(n)+1}}{f(n)+1} + sum_{k=1}^2 frac{B_k}{k!}n^{f(n)-k+1} cdotprod_{j=0}^{k-2}big(f(n)-jbig) + R_2\
%
&= frac{n^{f(n)+1}}{f(n)+1} + frac{(1 /2)}{1!}n^{f(n)-1+1} + frac{(1 /6)}{2!}n^{f(n)-2+1}f(n) + R_2\
%
&= n^{f(n)}left[frac{n}{f(n)+1} + frac{1}{2} + frac{1}{12}frac{f(n)}{n}right] + R_2\
end{align}



Example 1
For $f(n)=n$:
$$F_f(n)
= n^nleft[frac{n}{n+1} + frac{1}{2} + frac{1}{12}frac{n}{n}right] + R_2
= n^n left[frac{19}{12} - frac{1}{n+1}right] + R_2$$



$$|R_2| leq frac{1}{12} ncdot n^{n-1} = frac{1}{12} n^n$$
so
$$lim_{ntoinfty} frac{F_f(n)}{n^n} in left(tfrac{19}{12}-tfrac{1}{12}, tfrac{19}{12} + tfrac{1}{12}right) = left(tfrac{3}{2}, tfrac{5}{3}right)$$



Indeed, a check:
$$frac{e}{e-1} in (tfrac{3}{2}, tfrac{5}{3})$$



Example 2
For $f(n)=log(n)$:
begin{align}
F_f(n)
%
&= n^{log(n)}left[frac{n}{log(n)+1} + frac{1}{2} + frac{1}{12}frac{log(n)}{n}right] + R_2\
%
&= frac{n^{log(n)+1}}{log(n)}left[frac{log(n)}{log(n)+1} + frac{1}{2}frac{log(n)}{n} + frac{1}{12}left(frac{log(n)}{n}right)^2right] + R_2\
end{align}



begin{align}
|R_2|
&leq frac{1}{12} f(n) n^{f(n)-1}
= frac{1}{12} log(n) n^{log(n)-1}\
%
&= frac{1}{12} frac{n^{log(n)+1}}{log(n)} left(frac{log(n)}{n}right)^2
end{align}



So
$$lim_{ntoinfty} frac{F_f(n)}{frac{n^{log(n)+1}}{log(n)}} = 1$$
because $frac{|R_2|}{frac{n^{log(n)+1}}{log(n)}} = left(frac{log(n)}{n}right)^2 to 0$.



Example 3
$f(n) = n^alpha$ for $alphain(0,1)$:



begin{align}
F_f(n)
%
&=frac{n^{n^alpha+1}}{n^alpha+1} + frac{1}{2}n^{n^alpha-1+1} + frac{1}{12}n^{n^alpha-1}n^alpha + R_2\
%
&=frac{n^{alpha}}{n^alpha+1}n^{n^alpha+(1-alpha)} + frac{1}{2}n^{n^alpha} + frac{1}{12}n^{n^alpha-(1-alpha)} + R_2\
end{align}



$$|R_2|
%
leq frac{1}{12} f(n) n^{f(n)-1}
= frac{1}{12} n^{n^alpha-(1-alpha)}$$



$$lim_{ntoinfty} frac{F_f(n)}{n^{n^alpha+(1-alpha)}} = 1$$
as $$frac{|R_2|}{n^{n^alpha+(1-alpha)}} = frac{1}{12 n^{2(1-alpha)}} to 0$$



Example 4
For $f(n)=n^beta$, $beta > 1$:



begin{align}
F_f(n)
&=frac{n^{f(n)+1}}{f(n)}frac{f(n)}{f(n)+1} + frac{1}{2}n^{f(n)} + frac{1}{12}n^{f(n)-1}f(n) + R_2\
&= n^{n^beta - (beta - 1)} frac{n^beta}{n^beta+1} + frac{1}{2}n^{n^beta} + frac{1}{12}n^{n^beta+(beta-1)} + R_2
end{align}



$$|R_2|
leq frac{1}{12} n^beta n^{n^beta-1}
= frac{1}{12} n^{n^beta+(beta-1)}
$$



Hence
$$lim_{ntoinfty} frac{F_f(n)}{n^{n^beta+(beta-1)}}
in left(tfrac{1}{12} - tfrac{1}{12}, tfrac{1}{12} + tfrac{1}{12}right) = left(0, tfrac{1}{6}right)$$






share|cite|improve this answer











$endgroup$













  • $begingroup$
    Hi adfriedman, thank you for the detailed answer. Regarding your Example 4, I'm not sure you can conclude that the limit is in the open interval $(0, 1/6)$. In fact, I believe the limit will be 0 since heuristically it seems that $lim_{n to infty} F_f(n)/n^{n^{beta}} = 1$. In other words, the sum is dominated by just the last term.
    $endgroup$
    – maridia
    Dec 4 '18 at 15:59
















1












1








1





$begingroup$

(I have now rewritten this 3 times and, though another answer has been given in the meantime, I'll still submit it.)



If you are to use the Euler-Maclaurin formula you need to be careful with the remainder term, ensuring it doesn't blow up.



Let $g(x) = x^{f(n)}$, then we can write



begin{align}
F_f(n)
&equiv sum_{j=1}^n j^{f(n)}
= sum_{j=1}^n g(j)\
&= underbrace{int_0^n g(x);dx}_{(1)} + underbrace{sum_{k=1}^p frac{B_k}{k!}left(g^{(k-1)}(n) - g^{(k-1)}(0)right)}_{(2)} +underbrace{ R_p}_{(3)}
end{align}



(1)
$$int_0^n g(x);dx = int_0^n x^{f(n)};dx = frac{n^{f(n)+1}}{f(n)+1}$$



(2)
We can inductively show that $g^{(k)}(x) = P_k(n)x^{f(n)-k}$ for a sequence $big(P_k(n)big)_k$.
Clearly $g(x)=g^{(0)}(x)=P_0(n)x^{f(n) - 0}$ where $P_0(n)= 1$, and
begin{align}
g^{(k)}(x) &= frac{d}{dx} g^{(k-1)}(x)
= frac{d}{dx} P_{k-1}(n)x^{f(n)-(k-1)}\
&= P_{k-1}(n)big(f(n)-k+1big) x^{f(n)-(k-1)-1}
= P_k(n) x^{f(n)-k}
end{align}

where $P_k(n) equiv P_{k-1}(n)big(f(n)-k+1big)$. We can compute
$$P_k(n) = P_{k-1}(n)big(f(n)-k+1big) = prod_{j=0}^{k-1} big(f(n)-jbig)$$



Hence
begin{align}
sum_{k=1}^p frac{B_k}{k!}left(g^{(k-1)}(n) - g^{(k-1)}(0)right)
&= sum_{k=1}^p frac{B_k}{k!}left(P^{(k-1)}(n)n^{f(n)-k+1} - P^{(k-1)}(n)cdot 0right)\
&= sum_{k=1}^p frac{B_k}{k!}n^{f(n)-k+1} cdotprod_{j=0}^{k-2}big(f(n)-jbig)
end{align}



(3) We have the following bound on the remainder term:
begin{align}
|R_p|
&leq frac{2zeta(p )}{(2pi)^p} int_0^n |g^{(p )}(x)|;dx
= frac{2zeta(p )}{(2pi)^p} int_0^n |P_p(n)x^{f(n)-p}|;dx\
&overset{(*)}{=} frac{2zeta(p )}{(2pi)^p} |P_p(n)| frac{n^{f(n)-p+1}}{f(n)-p+1}
= frac{2zeta(p )}{(2pi)^p} prod_{j=0}^{p-2} big(f(n)-jbig) n^{f(n)-p+1}
end{align}



at $(*)$ we require $f(n)geq p$ for the integral to converge.



In particular, if we only take 2 terms from (2) (which we do to avoid $zeta(1)$), then we bound the



begin{align}
|R_2|
&leq frac{2zeta(2)}{(2pi)^2} prod_{j=0}^{2-2} big(f(n)-jbig) n^{f(n)-2+1}
= frac{2(pi^2/6)}{4pi^2} f(n) n^{f(n)-1}\
%
&= frac{1}{12} f(n) n^{f(n)-1}
end{align}



With $p=2$ we then write
begin{align}
F_f(n)
%
&= frac{n^{f(n)+1}}{f(n)+1} + sum_{k=1}^2 frac{B_k}{k!}n^{f(n)-k+1} cdotprod_{j=0}^{k-2}big(f(n)-jbig) + R_2\
%
&= frac{n^{f(n)+1}}{f(n)+1} + frac{(1 /2)}{1!}n^{f(n)-1+1} + frac{(1 /6)}{2!}n^{f(n)-2+1}f(n) + R_2\
%
&= n^{f(n)}left[frac{n}{f(n)+1} + frac{1}{2} + frac{1}{12}frac{f(n)}{n}right] + R_2\
end{align}



Example 1
For $f(n)=n$:
$$F_f(n)
= n^nleft[frac{n}{n+1} + frac{1}{2} + frac{1}{12}frac{n}{n}right] + R_2
= n^n left[frac{19}{12} - frac{1}{n+1}right] + R_2$$



$$|R_2| leq frac{1}{12} ncdot n^{n-1} = frac{1}{12} n^n$$
so
$$lim_{ntoinfty} frac{F_f(n)}{n^n} in left(tfrac{19}{12}-tfrac{1}{12}, tfrac{19}{12} + tfrac{1}{12}right) = left(tfrac{3}{2}, tfrac{5}{3}right)$$



Indeed, a check:
$$frac{e}{e-1} in (tfrac{3}{2}, tfrac{5}{3})$$



Example 2
For $f(n)=log(n)$:
begin{align}
F_f(n)
%
&= n^{log(n)}left[frac{n}{log(n)+1} + frac{1}{2} + frac{1}{12}frac{log(n)}{n}right] + R_2\
%
&= frac{n^{log(n)+1}}{log(n)}left[frac{log(n)}{log(n)+1} + frac{1}{2}frac{log(n)}{n} + frac{1}{12}left(frac{log(n)}{n}right)^2right] + R_2\
end{align}



begin{align}
|R_2|
&leq frac{1}{12} f(n) n^{f(n)-1}
= frac{1}{12} log(n) n^{log(n)-1}\
%
&= frac{1}{12} frac{n^{log(n)+1}}{log(n)} left(frac{log(n)}{n}right)^2
end{align}



So
$$lim_{ntoinfty} frac{F_f(n)}{frac{n^{log(n)+1}}{log(n)}} = 1$$
because $frac{|R_2|}{frac{n^{log(n)+1}}{log(n)}} = left(frac{log(n)}{n}right)^2 to 0$.



Example 3
$f(n) = n^alpha$ for $alphain(0,1)$:



begin{align}
F_f(n)
%
&=frac{n^{n^alpha+1}}{n^alpha+1} + frac{1}{2}n^{n^alpha-1+1} + frac{1}{12}n^{n^alpha-1}n^alpha + R_2\
%
&=frac{n^{alpha}}{n^alpha+1}n^{n^alpha+(1-alpha)} + frac{1}{2}n^{n^alpha} + frac{1}{12}n^{n^alpha-(1-alpha)} + R_2\
end{align}



$$|R_2|
%
leq frac{1}{12} f(n) n^{f(n)-1}
= frac{1}{12} n^{n^alpha-(1-alpha)}$$



$$lim_{ntoinfty} frac{F_f(n)}{n^{n^alpha+(1-alpha)}} = 1$$
as $$frac{|R_2|}{n^{n^alpha+(1-alpha)}} = frac{1}{12 n^{2(1-alpha)}} to 0$$



Example 4
For $f(n)=n^beta$, $beta > 1$:



begin{align}
F_f(n)
&=frac{n^{f(n)+1}}{f(n)}frac{f(n)}{f(n)+1} + frac{1}{2}n^{f(n)} + frac{1}{12}n^{f(n)-1}f(n) + R_2\
&= n^{n^beta - (beta - 1)} frac{n^beta}{n^beta+1} + frac{1}{2}n^{n^beta} + frac{1}{12}n^{n^beta+(beta-1)} + R_2
end{align}



$$|R_2|
leq frac{1}{12} n^beta n^{n^beta-1}
= frac{1}{12} n^{n^beta+(beta-1)}
$$



Hence
$$lim_{ntoinfty} frac{F_f(n)}{n^{n^beta+(beta-1)}}
in left(tfrac{1}{12} - tfrac{1}{12}, tfrac{1}{12} + tfrac{1}{12}right) = left(0, tfrac{1}{6}right)$$






share|cite|improve this answer











$endgroup$



(I have now rewritten this 3 times and, though another answer has been given in the meantime, I'll still submit it.)



If you are to use the Euler-Maclaurin formula you need to be careful with the remainder term, ensuring it doesn't blow up.



Let $g(x) = x^{f(n)}$, then we can write



begin{align}
F_f(n)
&equiv sum_{j=1}^n j^{f(n)}
= sum_{j=1}^n g(j)\
&= underbrace{int_0^n g(x);dx}_{(1)} + underbrace{sum_{k=1}^p frac{B_k}{k!}left(g^{(k-1)}(n) - g^{(k-1)}(0)right)}_{(2)} +underbrace{ R_p}_{(3)}
end{align}



(1)
$$int_0^n g(x);dx = int_0^n x^{f(n)};dx = frac{n^{f(n)+1}}{f(n)+1}$$



(2)
We can inductively show that $g^{(k)}(x) = P_k(n)x^{f(n)-k}$ for a sequence $big(P_k(n)big)_k$.
Clearly $g(x)=g^{(0)}(x)=P_0(n)x^{f(n) - 0}$ where $P_0(n)= 1$, and
begin{align}
g^{(k)}(x) &= frac{d}{dx} g^{(k-1)}(x)
= frac{d}{dx} P_{k-1}(n)x^{f(n)-(k-1)}\
&= P_{k-1}(n)big(f(n)-k+1big) x^{f(n)-(k-1)-1}
= P_k(n) x^{f(n)-k}
end{align}

where $P_k(n) equiv P_{k-1}(n)big(f(n)-k+1big)$. We can compute
$$P_k(n) = P_{k-1}(n)big(f(n)-k+1big) = prod_{j=0}^{k-1} big(f(n)-jbig)$$



Hence
begin{align}
sum_{k=1}^p frac{B_k}{k!}left(g^{(k-1)}(n) - g^{(k-1)}(0)right)
&= sum_{k=1}^p frac{B_k}{k!}left(P^{(k-1)}(n)n^{f(n)-k+1} - P^{(k-1)}(n)cdot 0right)\
&= sum_{k=1}^p frac{B_k}{k!}n^{f(n)-k+1} cdotprod_{j=0}^{k-2}big(f(n)-jbig)
end{align}



(3) We have the following bound on the remainder term:
begin{align}
|R_p|
&leq frac{2zeta(p )}{(2pi)^p} int_0^n |g^{(p )}(x)|;dx
= frac{2zeta(p )}{(2pi)^p} int_0^n |P_p(n)x^{f(n)-p}|;dx\
&overset{(*)}{=} frac{2zeta(p )}{(2pi)^p} |P_p(n)| frac{n^{f(n)-p+1}}{f(n)-p+1}
= frac{2zeta(p )}{(2pi)^p} prod_{j=0}^{p-2} big(f(n)-jbig) n^{f(n)-p+1}
end{align}



at $(*)$ we require $f(n)geq p$ for the integral to converge.



In particular, if we only take 2 terms from (2) (which we do to avoid $zeta(1)$), then we bound the



begin{align}
|R_2|
&leq frac{2zeta(2)}{(2pi)^2} prod_{j=0}^{2-2} big(f(n)-jbig) n^{f(n)-2+1}
= frac{2(pi^2/6)}{4pi^2} f(n) n^{f(n)-1}\
%
&= frac{1}{12} f(n) n^{f(n)-1}
end{align}



With $p=2$ we then write
begin{align}
F_f(n)
%
&= frac{n^{f(n)+1}}{f(n)+1} + sum_{k=1}^2 frac{B_k}{k!}n^{f(n)-k+1} cdotprod_{j=0}^{k-2}big(f(n)-jbig) + R_2\
%
&= frac{n^{f(n)+1}}{f(n)+1} + frac{(1 /2)}{1!}n^{f(n)-1+1} + frac{(1 /6)}{2!}n^{f(n)-2+1}f(n) + R_2\
%
&= n^{f(n)}left[frac{n}{f(n)+1} + frac{1}{2} + frac{1}{12}frac{f(n)}{n}right] + R_2\
end{align}



Example 1
For $f(n)=n$:
$$F_f(n)
= n^nleft[frac{n}{n+1} + frac{1}{2} + frac{1}{12}frac{n}{n}right] + R_2
= n^n left[frac{19}{12} - frac{1}{n+1}right] + R_2$$



$$|R_2| leq frac{1}{12} ncdot n^{n-1} = frac{1}{12} n^n$$
so
$$lim_{ntoinfty} frac{F_f(n)}{n^n} in left(tfrac{19}{12}-tfrac{1}{12}, tfrac{19}{12} + tfrac{1}{12}right) = left(tfrac{3}{2}, tfrac{5}{3}right)$$



Indeed, a check:
$$frac{e}{e-1} in (tfrac{3}{2}, tfrac{5}{3})$$



Example 2
For $f(n)=log(n)$:
begin{align}
F_f(n)
%
&= n^{log(n)}left[frac{n}{log(n)+1} + frac{1}{2} + frac{1}{12}frac{log(n)}{n}right] + R_2\
%
&= frac{n^{log(n)+1}}{log(n)}left[frac{log(n)}{log(n)+1} + frac{1}{2}frac{log(n)}{n} + frac{1}{12}left(frac{log(n)}{n}right)^2right] + R_2\
end{align}



begin{align}
|R_2|
&leq frac{1}{12} f(n) n^{f(n)-1}
= frac{1}{12} log(n) n^{log(n)-1}\
%
&= frac{1}{12} frac{n^{log(n)+1}}{log(n)} left(frac{log(n)}{n}right)^2
end{align}



So
$$lim_{ntoinfty} frac{F_f(n)}{frac{n^{log(n)+1}}{log(n)}} = 1$$
because $frac{|R_2|}{frac{n^{log(n)+1}}{log(n)}} = left(frac{log(n)}{n}right)^2 to 0$.



Example 3
$f(n) = n^alpha$ for $alphain(0,1)$:



begin{align}
F_f(n)
%
&=frac{n^{n^alpha+1}}{n^alpha+1} + frac{1}{2}n^{n^alpha-1+1} + frac{1}{12}n^{n^alpha-1}n^alpha + R_2\
%
&=frac{n^{alpha}}{n^alpha+1}n^{n^alpha+(1-alpha)} + frac{1}{2}n^{n^alpha} + frac{1}{12}n^{n^alpha-(1-alpha)} + R_2\
end{align}



$$|R_2|
%
leq frac{1}{12} f(n) n^{f(n)-1}
= frac{1}{12} n^{n^alpha-(1-alpha)}$$



$$lim_{ntoinfty} frac{F_f(n)}{n^{n^alpha+(1-alpha)}} = 1$$
as $$frac{|R_2|}{n^{n^alpha+(1-alpha)}} = frac{1}{12 n^{2(1-alpha)}} to 0$$



Example 4
For $f(n)=n^beta$, $beta > 1$:



begin{align}
F_f(n)
&=frac{n^{f(n)+1}}{f(n)}frac{f(n)}{f(n)+1} + frac{1}{2}n^{f(n)} + frac{1}{12}n^{f(n)-1}f(n) + R_2\
&= n^{n^beta - (beta - 1)} frac{n^beta}{n^beta+1} + frac{1}{2}n^{n^beta} + frac{1}{12}n^{n^beta+(beta-1)} + R_2
end{align}



$$|R_2|
leq frac{1}{12} n^beta n^{n^beta-1}
= frac{1}{12} n^{n^beta+(beta-1)}
$$



Hence
$$lim_{ntoinfty} frac{F_f(n)}{n^{n^beta+(beta-1)}}
in left(tfrac{1}{12} - tfrac{1}{12}, tfrac{1}{12} + tfrac{1}{12}right) = left(0, tfrac{1}{6}right)$$







share|cite|improve this answer














share|cite|improve this answer



share|cite|improve this answer








edited Dec 4 '18 at 9:33

























answered Dec 4 '18 at 9:24









adfriedmanadfriedman

3,161169




3,161169












  • $begingroup$
    Hi adfriedman, thank you for the detailed answer. Regarding your Example 4, I'm not sure you can conclude that the limit is in the open interval $(0, 1/6)$. In fact, I believe the limit will be 0 since heuristically it seems that $lim_{n to infty} F_f(n)/n^{n^{beta}} = 1$. In other words, the sum is dominated by just the last term.
    $endgroup$
    – maridia
    Dec 4 '18 at 15:59




















  • $begingroup$
    Hi adfriedman, thank you for the detailed answer. Regarding your Example 4, I'm not sure you can conclude that the limit is in the open interval $(0, 1/6)$. In fact, I believe the limit will be 0 since heuristically it seems that $lim_{n to infty} F_f(n)/n^{n^{beta}} = 1$. In other words, the sum is dominated by just the last term.
    $endgroup$
    – maridia
    Dec 4 '18 at 15:59


















$begingroup$
Hi adfriedman, thank you for the detailed answer. Regarding your Example 4, I'm not sure you can conclude that the limit is in the open interval $(0, 1/6)$. In fact, I believe the limit will be 0 since heuristically it seems that $lim_{n to infty} F_f(n)/n^{n^{beta}} = 1$. In other words, the sum is dominated by just the last term.
$endgroup$
– maridia
Dec 4 '18 at 15:59






$begingroup$
Hi adfriedman, thank you for the detailed answer. Regarding your Example 4, I'm not sure you can conclude that the limit is in the open interval $(0, 1/6)$. In fact, I believe the limit will be 0 since heuristically it seems that $lim_{n to infty} F_f(n)/n^{n^{beta}} = 1$. In other words, the sum is dominated by just the last term.
$endgroup$
– maridia
Dec 4 '18 at 15:59




















draft saved

draft discarded




















































Thanks for contributing an answer to Mathematics Stack Exchange!


  • Please be sure to answer the question. Provide details and share your research!

But avoid



  • Asking for help, clarification, or responding to other answers.

  • Making statements based on opinion; back them up with references or personal experience.


Use MathJax to format equations. MathJax reference.


To learn more, see our tips on writing great answers.




draft saved


draft discarded














StackExchange.ready(
function () {
StackExchange.openid.initPostLogin('.new-post-login', 'https%3a%2f%2fmath.stackexchange.com%2fquestions%2f3024800%2fasymptotic-of-sum-sum-j-1n-jfn%23new-answer', 'question_page');
}
);

Post as a guest















Required, but never shown





















































Required, but never shown














Required, but never shown












Required, but never shown







Required, but never shown

































Required, but never shown














Required, but never shown












Required, but never shown







Required, but never shown







Popular posts from this blog

How do I know what Microsoft account the skydrive app is syncing to?

When does type information flow backwards in C++?

Grease: Live!